How one can write the delta Dirac function using Fourier series expansion such that summation is performed over odd indices only?

The name of the pictureThe name of the pictureThe name of the pictureClash Royale CLAN TAG#URR8PPP











up vote
1
down vote

favorite












The delta Dirac function can be presented in the form of Fourier series expansion as
$$
delta(x)=frac12pi
+frac1pi sum_nge 1 cos (nx) , .
$$
The proof is straightforward and can be found e.g. in this Mathworld Wolfram article.



In a mathematical physics problem, I would like to make use of the Fourier series technique in order to solve a system of differential equations involving a delta Dirac function, such that the solution has the form
$$
f(x) = sum_nge 1 f_n cos left( (2n-1)x right) , .
$$
That is, only the odd indices have to be need to be included in the Fourier series representation of the delta Dirac function.



What I tries is:
$$
delta(x) = frac2pi sum_nge 1 cos left( (2n-1)x right) , ,
$$
which seems to lead to correct final results.
I am wondering whether this can be proved in a rigorous way and/or whether this is true in the first place. Any help or comment would be very helpful.



Mickhausen







share|cite|improve this question

















  • 1




    How rigorous do you mean? Are you willing to delve into the theory of distributions?
    – Jason Born
    Jul 18 at 12:06










  • @JasonBorn A basic demonstration using the standard analytical tools would be highly appreciated. The expression seems to work out nicely but i unfortunately do not know why..
    – Mickhausen St Ilgen Sandhausen
    Jul 18 at 12:14














up vote
1
down vote

favorite












The delta Dirac function can be presented in the form of Fourier series expansion as
$$
delta(x)=frac12pi
+frac1pi sum_nge 1 cos (nx) , .
$$
The proof is straightforward and can be found e.g. in this Mathworld Wolfram article.



In a mathematical physics problem, I would like to make use of the Fourier series technique in order to solve a system of differential equations involving a delta Dirac function, such that the solution has the form
$$
f(x) = sum_nge 1 f_n cos left( (2n-1)x right) , .
$$
That is, only the odd indices have to be need to be included in the Fourier series representation of the delta Dirac function.



What I tries is:
$$
delta(x) = frac2pi sum_nge 1 cos left( (2n-1)x right) , ,
$$
which seems to lead to correct final results.
I am wondering whether this can be proved in a rigorous way and/or whether this is true in the first place. Any help or comment would be very helpful.



Mickhausen







share|cite|improve this question

















  • 1




    How rigorous do you mean? Are you willing to delve into the theory of distributions?
    – Jason Born
    Jul 18 at 12:06










  • @JasonBorn A basic demonstration using the standard analytical tools would be highly appreciated. The expression seems to work out nicely but i unfortunately do not know why..
    – Mickhausen St Ilgen Sandhausen
    Jul 18 at 12:14












up vote
1
down vote

favorite









up vote
1
down vote

favorite











The delta Dirac function can be presented in the form of Fourier series expansion as
$$
delta(x)=frac12pi
+frac1pi sum_nge 1 cos (nx) , .
$$
The proof is straightforward and can be found e.g. in this Mathworld Wolfram article.



In a mathematical physics problem, I would like to make use of the Fourier series technique in order to solve a system of differential equations involving a delta Dirac function, such that the solution has the form
$$
f(x) = sum_nge 1 f_n cos left( (2n-1)x right) , .
$$
That is, only the odd indices have to be need to be included in the Fourier series representation of the delta Dirac function.



What I tries is:
$$
delta(x) = frac2pi sum_nge 1 cos left( (2n-1)x right) , ,
$$
which seems to lead to correct final results.
I am wondering whether this can be proved in a rigorous way and/or whether this is true in the first place. Any help or comment would be very helpful.



Mickhausen







share|cite|improve this question













The delta Dirac function can be presented in the form of Fourier series expansion as
$$
delta(x)=frac12pi
+frac1pi sum_nge 1 cos (nx) , .
$$
The proof is straightforward and can be found e.g. in this Mathworld Wolfram article.



In a mathematical physics problem, I would like to make use of the Fourier series technique in order to solve a system of differential equations involving a delta Dirac function, such that the solution has the form
$$
f(x) = sum_nge 1 f_n cos left( (2n-1)x right) , .
$$
That is, only the odd indices have to be need to be included in the Fourier series representation of the delta Dirac function.



What I tries is:
$$
delta(x) = frac2pi sum_nge 1 cos left( (2n-1)x right) , ,
$$
which seems to lead to correct final results.
I am wondering whether this can be proved in a rigorous way and/or whether this is true in the first place. Any help or comment would be very helpful.



Mickhausen









share|cite|improve this question












share|cite|improve this question




share|cite|improve this question








edited Jul 18 at 11:57
























asked Jul 18 at 10:52









Mickhausen St Ilgen Sandhausen

92419




92419







  • 1




    How rigorous do you mean? Are you willing to delve into the theory of distributions?
    – Jason Born
    Jul 18 at 12:06










  • @JasonBorn A basic demonstration using the standard analytical tools would be highly appreciated. The expression seems to work out nicely but i unfortunately do not know why..
    – Mickhausen St Ilgen Sandhausen
    Jul 18 at 12:14












  • 1




    How rigorous do you mean? Are you willing to delve into the theory of distributions?
    – Jason Born
    Jul 18 at 12:06










  • @JasonBorn A basic demonstration using the standard analytical tools would be highly appreciated. The expression seems to work out nicely but i unfortunately do not know why..
    – Mickhausen St Ilgen Sandhausen
    Jul 18 at 12:14







1




1




How rigorous do you mean? Are you willing to delve into the theory of distributions?
– Jason Born
Jul 18 at 12:06




How rigorous do you mean? Are you willing to delve into the theory of distributions?
– Jason Born
Jul 18 at 12:06












@JasonBorn A basic demonstration using the standard analytical tools would be highly appreciated. The expression seems to work out nicely but i unfortunately do not know why..
– Mickhausen St Ilgen Sandhausen
Jul 18 at 12:14




@JasonBorn A basic demonstration using the standard analytical tools would be highly appreciated. The expression seems to work out nicely but i unfortunately do not know why..
– Mickhausen St Ilgen Sandhausen
Jul 18 at 12:14










2 Answers
2






active

oldest

votes

















up vote
1
down vote



accepted










The details below are a bit of a quick and dirty (more physicist approach) and that it ought to be demonstrated properly by distribution theory.



The expansion that you use is not a delta function but a combination thereof. I.e., it is not to difficult to check that:
$$
delta(x) - delta(x-pi) = frac2pi sum_n ~textodd cos( n x)
$$
Note that the series expansion implies that you consider functions with a $2pi$-periodicity, and that one also could have written it in a more symmetric form as
$$
delta(x) - frac12 left[ delta(x-pi) + delta(x+pi)right]
$$
that again should be repeated periodically.



Checking the validity of the expansion needs a a little extra attention because the second delta peak is located at the boundary of the integration interval for the coefficients. One simple approach to circumvent this issue is to consider the complex series:
$$
f(x) = sum_n A_n e^imath n x
$$
with
$$
A_n = frac12 pi int_-pi/2^3 pi/2 f(x) e^-imath n x textd x
$$
which would not affect the usual definition for an arbitrary function $f(x)$, but here ensures that both delta functions are within the integration interval. As an alternative approach one could count the delta functions only half if they are exactly on the boundaries.



Whether your approach is valid or not depends on the actual problem. It could be accidental of course, but I would not be surprised if the the underlying symmetry of the problem indeed allows for it.






share|cite|improve this answer




























    up vote
    1
    down vote













    No, that can't be right. The Fourier series of a distribution is unique.






    share|cite|improve this answer





















      Your Answer




      StackExchange.ifUsing("editor", function ()
      return StackExchange.using("mathjaxEditing", function ()
      StackExchange.MarkdownEditor.creationCallbacks.add(function (editor, postfix)
      StackExchange.mathjaxEditing.prepareWmdForMathJax(editor, postfix, [["$", "$"], ["\\(","\\)"]]);
      );
      );
      , "mathjax-editing");

      StackExchange.ready(function()
      var channelOptions =
      tags: "".split(" "),
      id: "69"
      ;
      initTagRenderer("".split(" "), "".split(" "), channelOptions);

      StackExchange.using("externalEditor", function()
      // Have to fire editor after snippets, if snippets enabled
      if (StackExchange.settings.snippets.snippetsEnabled)
      StackExchange.using("snippets", function()
      createEditor();
      );

      else
      createEditor();

      );

      function createEditor()
      StackExchange.prepareEditor(
      heartbeatType: 'answer',
      convertImagesToLinks: true,
      noModals: false,
      showLowRepImageUploadWarning: true,
      reputationToPostImages: 10,
      bindNavPrevention: true,
      postfix: "",
      noCode: true, onDemand: true,
      discardSelector: ".discard-answer"
      ,immediatelyShowMarkdownHelp:true
      );



      );








       

      draft saved


      draft discarded


















      StackExchange.ready(
      function ()
      StackExchange.openid.initPostLogin('.new-post-login', 'https%3a%2f%2fmath.stackexchange.com%2fquestions%2f2855464%2fhow-one-can-write-the-delta-dirac-function-using-fourier-series-expansion-such-t%23new-answer', 'question_page');

      );

      Post as a guest






























      2 Answers
      2






      active

      oldest

      votes








      2 Answers
      2






      active

      oldest

      votes









      active

      oldest

      votes






      active

      oldest

      votes








      up vote
      1
      down vote



      accepted










      The details below are a bit of a quick and dirty (more physicist approach) and that it ought to be demonstrated properly by distribution theory.



      The expansion that you use is not a delta function but a combination thereof. I.e., it is not to difficult to check that:
      $$
      delta(x) - delta(x-pi) = frac2pi sum_n ~textodd cos( n x)
      $$
      Note that the series expansion implies that you consider functions with a $2pi$-periodicity, and that one also could have written it in a more symmetric form as
      $$
      delta(x) - frac12 left[ delta(x-pi) + delta(x+pi)right]
      $$
      that again should be repeated periodically.



      Checking the validity of the expansion needs a a little extra attention because the second delta peak is located at the boundary of the integration interval for the coefficients. One simple approach to circumvent this issue is to consider the complex series:
      $$
      f(x) = sum_n A_n e^imath n x
      $$
      with
      $$
      A_n = frac12 pi int_-pi/2^3 pi/2 f(x) e^-imath n x textd x
      $$
      which would not affect the usual definition for an arbitrary function $f(x)$, but here ensures that both delta functions are within the integration interval. As an alternative approach one could count the delta functions only half if they are exactly on the boundaries.



      Whether your approach is valid or not depends on the actual problem. It could be accidental of course, but I would not be surprised if the the underlying symmetry of the problem indeed allows for it.






      share|cite|improve this answer

























        up vote
        1
        down vote



        accepted










        The details below are a bit of a quick and dirty (more physicist approach) and that it ought to be demonstrated properly by distribution theory.



        The expansion that you use is not a delta function but a combination thereof. I.e., it is not to difficult to check that:
        $$
        delta(x) - delta(x-pi) = frac2pi sum_n ~textodd cos( n x)
        $$
        Note that the series expansion implies that you consider functions with a $2pi$-periodicity, and that one also could have written it in a more symmetric form as
        $$
        delta(x) - frac12 left[ delta(x-pi) + delta(x+pi)right]
        $$
        that again should be repeated periodically.



        Checking the validity of the expansion needs a a little extra attention because the second delta peak is located at the boundary of the integration interval for the coefficients. One simple approach to circumvent this issue is to consider the complex series:
        $$
        f(x) = sum_n A_n e^imath n x
        $$
        with
        $$
        A_n = frac12 pi int_-pi/2^3 pi/2 f(x) e^-imath n x textd x
        $$
        which would not affect the usual definition for an arbitrary function $f(x)$, but here ensures that both delta functions are within the integration interval. As an alternative approach one could count the delta functions only half if they are exactly on the boundaries.



        Whether your approach is valid or not depends on the actual problem. It could be accidental of course, but I would not be surprised if the the underlying symmetry of the problem indeed allows for it.






        share|cite|improve this answer























          up vote
          1
          down vote



          accepted







          up vote
          1
          down vote



          accepted






          The details below are a bit of a quick and dirty (more physicist approach) and that it ought to be demonstrated properly by distribution theory.



          The expansion that you use is not a delta function but a combination thereof. I.e., it is not to difficult to check that:
          $$
          delta(x) - delta(x-pi) = frac2pi sum_n ~textodd cos( n x)
          $$
          Note that the series expansion implies that you consider functions with a $2pi$-periodicity, and that one also could have written it in a more symmetric form as
          $$
          delta(x) - frac12 left[ delta(x-pi) + delta(x+pi)right]
          $$
          that again should be repeated periodically.



          Checking the validity of the expansion needs a a little extra attention because the second delta peak is located at the boundary of the integration interval for the coefficients. One simple approach to circumvent this issue is to consider the complex series:
          $$
          f(x) = sum_n A_n e^imath n x
          $$
          with
          $$
          A_n = frac12 pi int_-pi/2^3 pi/2 f(x) e^-imath n x textd x
          $$
          which would not affect the usual definition for an arbitrary function $f(x)$, but here ensures that both delta functions are within the integration interval. As an alternative approach one could count the delta functions only half if they are exactly on the boundaries.



          Whether your approach is valid or not depends on the actual problem. It could be accidental of course, but I would not be surprised if the the underlying symmetry of the problem indeed allows for it.






          share|cite|improve this answer













          The details below are a bit of a quick and dirty (more physicist approach) and that it ought to be demonstrated properly by distribution theory.



          The expansion that you use is not a delta function but a combination thereof. I.e., it is not to difficult to check that:
          $$
          delta(x) - delta(x-pi) = frac2pi sum_n ~textodd cos( n x)
          $$
          Note that the series expansion implies that you consider functions with a $2pi$-periodicity, and that one also could have written it in a more symmetric form as
          $$
          delta(x) - frac12 left[ delta(x-pi) + delta(x+pi)right]
          $$
          that again should be repeated periodically.



          Checking the validity of the expansion needs a a little extra attention because the second delta peak is located at the boundary of the integration interval for the coefficients. One simple approach to circumvent this issue is to consider the complex series:
          $$
          f(x) = sum_n A_n e^imath n x
          $$
          with
          $$
          A_n = frac12 pi int_-pi/2^3 pi/2 f(x) e^-imath n x textd x
          $$
          which would not affect the usual definition for an arbitrary function $f(x)$, but here ensures that both delta functions are within the integration interval. As an alternative approach one could count the delta functions only half if they are exactly on the boundaries.



          Whether your approach is valid or not depends on the actual problem. It could be accidental of course, but I would not be surprised if the the underlying symmetry of the problem indeed allows for it.







          share|cite|improve this answer













          share|cite|improve this answer



          share|cite|improve this answer











          answered Jul 18 at 15:15









          Ronald Blaak

          1,69438




          1,69438




















              up vote
              1
              down vote













              No, that can't be right. The Fourier series of a distribution is unique.






              share|cite|improve this answer

























                up vote
                1
                down vote













                No, that can't be right. The Fourier series of a distribution is unique.






                share|cite|improve this answer























                  up vote
                  1
                  down vote










                  up vote
                  1
                  down vote









                  No, that can't be right. The Fourier series of a distribution is unique.






                  share|cite|improve this answer













                  No, that can't be right. The Fourier series of a distribution is unique.







                  share|cite|improve this answer













                  share|cite|improve this answer



                  share|cite|improve this answer











                  answered Jul 18 at 15:30









                  David C. Ullrich

                  54.3k33583




                  54.3k33583






















                       

                      draft saved


                      draft discarded


























                       


                      draft saved


                      draft discarded














                      StackExchange.ready(
                      function ()
                      StackExchange.openid.initPostLogin('.new-post-login', 'https%3a%2f%2fmath.stackexchange.com%2fquestions%2f2855464%2fhow-one-can-write-the-delta-dirac-function-using-fourier-series-expansion-such-t%23new-answer', 'question_page');

                      );

                      Post as a guest













































































                      Comments

                      Popular posts from this blog

                      Color the edges and diagonals of a regular polygon

                      Relationship between determinant of matrix and determinant of adjoint?

                      What is the equation of a 3D cone with generalised tilt?